User avatar
 
bbirdwell
Thanks Received: 864
Atticus Finch
Atticus Finch
 
Posts: 803
Joined: April 16th, 2009
 
This post thanked 3 times.
 
 

Q23 - Candidate: The government spends $500

by bbirdwell Fri Feb 26, 2010 7:27 pm

I think this is a tough problem, in part because it's hard to pin down "the logical flaw" in a nice, neat package like you can on other arguments. When you encounter a situation like that, especially on a "parallel" question, I think it's important to map out the reasoning than it is to name a certain flaw. Here's how I thought about it.

govt spends more money on X than Y.
more ppl die from Y than X

govt would save lives by spending less money on X and more on Y.

The flaws in this construction are many -- the assumption that spending more money will save lives is the biggest. But again, I think on a question like this, just mapping the reasoning out is usually sufficient to answer the question.

(A) govt enforces X more closely than Y
more people die on X than Y

govt would save lives by spending less money on X and more on Y.
Notice the difference in the pattern here. Also, this answer choice is trying to make you fall for the trap of picking it because it has the same content as the argument above.

(B) musician spends more time on X than Y
she is hired more often for Y than X

she would get hired more by spending less hours on X and more on Y
Yep. This is a match.

(C) autos burn more gas/minute on X than on Y
autos get fewer miles/gallon on Y than on X

therefore gas would be saved by driving less on Y and more on X.
Not a match.

(D) team spends more hours on X than Y
times for Y are better than X

would increase wins if it spent less time X and more on Y.
Tempting choice, and the second best match, in my opinion. Subtle difference in that this choice makes an additional leap in logic that the original does not. Namely, the leap from being faster at one stroke than another to "winning more swim meets."

(E) banks have higher profits on loans X than on Y loans.
borrowers willing to borrow larger sums with Y rates than X rates.

banks would make higher profits if they gave more Y loans and fewer X loans.
This is not a very good match, either.

(B) is the cleanest match, and therefore the best answer.
I host free online workshop/Q&A sessions called Zen and the Art of LSAT. You can find upcoming dates here: http://www.manhattanlsat.com/zen-and-the-art.cfm
 
skapur777
Thanks Received: 6
Atticus Finch
Atticus Finch
 
Posts: 145
Joined: March 27th, 2011
 
 
 

Re: Q23 - Candidate: The government spends $500

by skapur777 Sun Apr 24, 2011 10:36 pm

Why is E wrong? I got this question right but is it wrong just beacuse thof the line 'if they gave more loans at low rates and fewer loans at high rates'? Because they reversed the order?

Because the original says shift funds from X to Y, and this one says 'give more loans to Y than X', is it really just because the letters have been switched?
User avatar
 
bbirdwell
Thanks Received: 864
Atticus Finch
Atticus Finch
 
Posts: 803
Joined: April 16th, 2009
 
 
 

Re: PT 23, Section 3, #23; Candidate: The government spends $500

by bbirdwell Sat Apr 30, 2011 6:18 pm

No, it's more about the concept. In the first argument, we have information about spending (x vs y) in the first sentence, and we make a conclusion about spending (x vs y).

The same is true for the correct answer, (B). We have info about practice hours (x vs y), we make a conclusion about practice hours (x vs y).

In (E), we have info about profit margins in the first sentence, and we make a conclusion about the number of loans.

See what I mean? In the original and (B), we essentially have "right now, we're doing more x than y. we should change this."

In (E), we don't know how many of each type of loan we're currently giving. The first sentence does not give us the same kind of information that the first sentence in the original does.

In order to be correct, (E) would have to say something more like "Banks give more high-interest loans than low-interest loans. But people are willing to borrow larger sums at low rates. Therefore, banks would be more profitable if they gave more low-rate loans and fewer high-rate loans."
I host free online workshop/Q&A sessions called Zen and the Art of LSAT. You can find upcoming dates here: http://www.manhattanlsat.com/zen-and-the-art.cfm
 
jionggangtu
Thanks Received: 0
Forum Guests
 
Posts: 21
Joined: February 20th, 2012
 
 
 

Re: Q23 - Candidate: The government spends $500

by jionggangtu Thu May 10, 2012 6:42 pm

I still don't get why D is wrong.

Isn't D saying this:

1) Swiming Team spends more time on X than Y.

2) Swiming Team is better at Y than X.

3) Swiming Team's winning opportunity will increase if swtiching practising focus/time from X to Y?
 
anjelica.grace
Thanks Received: 5
Jackie Chiles
Jackie Chiles
 
Posts: 41
Joined: November 17th, 2011
 
This post thanked 1 time.
 
 

Re: Q23 - Candidate: The government spends $500

by anjelica.grace Tue Aug 21, 2012 3:29 am

jionggangtu Wrote:I still don't get why D is wrong.

Isn't D saying this:

1) Swiming Team spends more time on X than Y.

2) Swiming Team is better at Y than X.

3) Swiming Team's winning opportunity will increase if swtiching practising focus/time from X to Y?


If you're satisfied with a quick explanation as to why (D) is wrong: it shifts scope from "better times" to "winning meets," unlike the original ("more people die" and "more lives saved") and the correct choice ("hired more often" and "increase the number of playing engagements"), where scope shift is not the problem.

If you don't mind a long explanation for why I think (D) is primarily wrong, then it's important to first see how the stimulus and correct choice are parallel:


-----------------------------------------

ORIGINAL [X = highway safety, Y = anti-smoking]
The government spends much more money on X than it spends on Y.
But more people die from Y than die from X.
So, the government would save more lives if it spent more money on Y rather than X.

X receives more resources than Y.
Y takes priority over X.
Thus, shifting resources from X (smaller priority) to Y (bigger priority) will cause the desired result (more lives saved).

ANSWER CHOICE B - CORRECT [X = guitar, Y = saxophone]
A musician spends many more hours practicing X than she spends practicing Y.
But she is hired much more often for Y than for X.
So, the musician would increase her number of gigs if she spent more time practicing Y rather than X.

X receives more resources than Y.
Y takes priority over X.
Thus, shifting resources from X (smaller priority) to Y (bigger priority) will cause the desired result (more playing engagements).

-----------------------------------------

ABSTRACTION

The current ratio of resources is laid out.
An incentive to reverse that ratio is offered based on the current distribution of effects/results.
Thus, the desired result would occur if the ratio of resources was reversed.

I highlighted what I believe is the key piece of evidence that leads us to identify the flaw. That is, there is an interest to switch resources.

FLAW: assuming that the resources are effective (i.e., that the resource will cause the desired result).
Just because there is a stronger incentive to switch resources doesn't mean the incentive is actually rewarded when the resources are switched.

I tried mapping it out algebraically but then it yielded an incorrect answer choice, so I realized I had to break this down more conceptually and characterize the argument rather than simply mapping it out (I personally prefer mapping it out, but I don't think this question in particular lends itself to such algebraic representation. You get lost if you don't grasp the core.). After literally hours of analysis, I have come to my own conclusion that while there is an element of causation in this question, correlation, despite what we often see, is not invoked in the premises. If there was correlation, then (B) would not be the correct choice. Sure, we can say that in the premises of the stimulus, more funds correlate with more lives saved, but if we say that more hours of practice correlate with more playing engagements, as described in (B), then wouldn't we expect the musician to be getting more guitar gigs since she was devoting more practice time towards the guitar? That's where the original and correct choice differ, so that cannot be the parallel component. What is parallel is what I highlighted, that consideration in favor of switching. I don't know how else to characterize it, motive, interest, advantage, etc. -- in both cases, a pressing need/desire for switching is described, and in both cases, the conclusion hastily assumes that need/desire would be met.

--------------------------------------------

Now why ANSWER CHOICE D - INCORRECT [X = backstroke, Y = breaststroke]
The local swim team spends many more hours practicing X than it spends practicing Y.
But the team's lap times for Y are much better than X.
So, the team would win more meets if it spent more time practicing Y rather than X.

X receives more resources than Y.
X takes priority over Y.
Thus, resources should shift from X (bigger priority) to Y (smaller priority).

HERE, THE DIFFERENCE (AND THUS WHY D IS NOT PARALLEL) IS THAT THERE IS NO INCENTIVE TO SWITCH.

In fact, there is an incentive to maintain the current ratio of resources. If anything, switching just goes against common sense. If you're spending more time on the backstroke and your times are worse for the backstroke, why switch efforts to something you're already good at? (Unless, of course, you give up and are trying to cut your losses, but that's a whole different issue.) At any rate, it never gets to the point of assuming a switch in resources will solve the problem (i.e., meet the needs of the swim team).

Because of this structural disparity from the original, (D) cannot assume that resources are effective, since switching resources in this case would be counterproductive. On the other hand, if (D) had made the same faulty assumption that the original makes, i.e. that resources would succeed in effecting the desired results, then practice times would certainly not be proposed to change.

Note: this is also the reason why (A) is wrong, there is no incentive to switch resources from freeways to tollways if where more resources are currently going (i.e. freeways) is where more people are dying.

--------------------------------------------

If someone can simplify my reasoning (if it's even correct), that would be great. This was, by far, the hardest LR question I've come across, and I was determined to get to the bottom of it. So, sorry for how long this is.